Matemática, perguntado por Usuário anônimo, 3 meses atrás

Determinar a terceira coordenada do vetor u de tamanho Raíz de 2, que seja simultaneamente ortogonal ao vetor a =(1,−1,1) e ao vetor b =(1,1,1), além disso u forma um ângulo agudo (ângulo menor que 90 graus) com o vetor de coordenadas (2,3,-4).


Vicktoras: Encontrar a terceira coordenada ou as 3 coordenadas?

Soluções para a tarefa

Respondido por Vicktoras
2

Pelos cálculos realizado, chegamos a conclusão de que vetor (u) é dado por \bf u = (1,0,-1)

Explicação

Pela questão, sabemos algumas informações sobre o vetor (u), como perpendicularidade com outros vetores e módulo.

  • Módulo:

Primeiramente vamos utilizar esta informação sobre o módulo, já que para determinar a magnitude de um vetor basta retirar a raiz do quadrado dos termos.

 | |w| |  =  \sqrt{(w_{x}) {}^{2}   +(w_{y}) {}^{2}  +  \cdots +(w_{n}) {}^{2} }  \\

Como não sabemos as coordenadas do vetor (u), vamos supor um vetor de coordenadas genéricas. Tomemos então u=(x,\:y,\:z). Logo, o seu módulo será igual a:

 \:  \:  \:  \:  \:  \:  \:  \:  \: | |u| |  =  \sqrt{x {}^{2} + y {}^{2} + z {}^{2}   }

No enunciado é dito que o módulo de u é igual a raiz de 2. Substituindo esta informação:

 | |u| |  = \sqrt{x {}^{2}  + y {}^{2}  + z {}^{2} } \:   \to \:   \sqrt{2}  =  \sqrt{x {}^{2} + y {}^{2}   + z {}^{2} }   \:  \:  \:  \:  \:  \:  \:  \:  \:  \: \\  \\  ( \sqrt{2}  ) {}^{2}   = ( \sqrt{x {}^{2}  + y {}^{2}  + z {}^{2} } ) { }^{2} \:  \: \to \:  \:   \boxed{x {}^{2}  + y {}^{2}  + z {}^{2}  = 2 \:  \: (I)}

  • Perpendicularidade:

Um vetor ser perpendicular a outro nos permite aplicar uma propriedade do produto escalar, que fala que se houver perpendicularidade entre dois vetores, o produto escalar entre eles é igual a 0.

 u\:\cdot\:v=0 se e somente se o ângulo entre eles é igual a  90^o.

\large \ast Lembrando que o produto escalar é dado pela soma da multiplicação de coordenada a coordenada dos vetores envolvidos.

 \bf{produto \: escalar} \\ \begin{cases}u=(u_x,\:u_y,\:u_z)\:\: e\:\: v=(v_x,\:v_y,\:v_z)\\ \\u\cdot v=(u_x\:.v_x+u_y\:.\:v_y+u_z\:.\:v_z)\end{cases}

  • Produto escalar entre u e a:

Sendo  u=(x,\:y,\:z) e  a=(1,\:-1,\:1) , como eles são ortogonais, então:

 u \:  \cdot \: a = 0 \:  \:  \to \:  \:   (x,\:y,\:z)   \:  \cdot \:(1,\: - 1,\:1) = 0 \\ \\  x \: . \: 1 + y \: . \: ( - 1) +   z  \: . \: 1 = 0  \:  \:  \to \:  \:  \boxed{x - y + z = 0 \:  \:II)}  \\

  • Produto escalar entre u e b:

Sendo  u=(x,\:y,\:z) e  b=(1,\:1,\:1).

u \:  \cdot \: b = 0  \:   \: \to \:  \:   (x,\:y,\:z)   \:  \cdot \:(1,\:  1,\:1) = 0 \\ \\  x \: . \: 1 + y \: . \: 1+   z  \: . \: 1 = 0  \:  \:  \to \:  \:  \boxed{x  +  y + z = 0 \:  \: I II)}  \\

  • Sistema de equações:

Observe que três relações de três incógnitas foram montadas, ou seja, podemos montar um sistema (3 x 3) com estas equações e resolvê-lo para encontrar o valor de cada incógnita.

 \: \:  \:  \:   \:  \:  \:  \:  \:  \:  \:  \:  \:   \: \begin{cases}x + y + z = 0 \\ x - y + z = 0 \\ x {}^{2} + y {}^{2}   + z {}^{2}   = 2\end{cases}

Para resolver este sistema, vamos utilizar o método da substituição. Iniciando por:

(1)  \begin{cases}x -  y + z = 0  \\   x=  y - z \:  \:  \end{cases} \:  \: (2) \begin{cases}x + y + z = 0 \\ y - z + y + z = 0 \\ y = 0 \end{cases} \\

Como y = 0, obtemos que x e z possuem dois valores distintos, sendo elas:  x=\pm1 e  z = \pm1

Já que estamos calculando as coordenadas do vetor (u) e obtemos dois possíveis valores, ficando com duas possibilidades para o vetor (u), que são:  \: u_1 = (1,0,-1)  \:  \:  \: e \:  \:  \:  u_2 = (-1,0,1) \\

  • Ângulo entre vetores:

Para de fato determinar qual o vetor certo que cumpra as exigências do enunciado, vamos utilizar a última informação que nos resta, que é a de que o vetor (u) forma um ângulo agudo com o vetor j = (2,3,-4) . O cálculo dará-se através da fórmula do ângulo entre dois vetores, sendo ela: \cos( \theta) =  \frac{u \:  \cdot \: v}{ | |u| |  \: . \:  | |v| | }, \: 0 \leqslant  \theta \leqslant 180 {}^{o}  \\  . Agora vamos calcular cada ângulo separadamente.

  • ângulo entre  \bf u_1e\bf j :

 \cos( \theta) =  \frac{(1,\:0,\:  - 1) \:  \cdot \:(2,\:3,\: - 4)}{ \sqrt{1 {}^{2}  + 0 {}^{2} + 1 {}^{2}   } \:  \: .  \:  \:  \sqrt{2 {}^{2}  + 3 {}^{2}  + 4 {}^{2} } }  \\  \\  \cos( \theta) =  \frac{1.2 \:  +  \: 0.3 \: +  \: ( - 1).( - 4) }{ \sqrt{2}  \:  \: . \:  \:  \sqrt{29} }  \\  \\  \cos( \theta) =  \frac{6}{ \sqrt{48} }  \:  \:  \to \:  \:  \theta =  \arccos \left(  \frac{6}{ \sqrt{48} } \right) \\  \\   \boxed{\theta =  \frac{\pi}{6}  \:  \:  \: ou \:  \:  30 {}^{o} }

  • ângulo entre \bf u_2e\bf j :

 \cos( \theta) =  \frac{( - 1,\:0,\:  1) \:  \cdot \:(2,\:3,\: - 4)}{ \sqrt{1 {}^{2}  + 0 {}^{2} + 1 {}^{2}   } \:  \: .  \:  \:  \sqrt{2 {}^{2}  + 3 {}^{2}  + 4 {}^{2} } }  \\  \\  \cos( \theta) =  \frac{( - 1).2 \:  +  \: 0.3 \: +  \:  1.( - 4) }{ \sqrt{2}  \:  \: . \:  \:  \sqrt{29} }  \\  \\  \cos( \theta) =   - \frac{6}{ \sqrt{48} }  \:  \:  \to \:  \:  \theta =   \arccos \left(   - \frac{6}{ \sqrt{48} } \right) \\  \\   \boxed{\theta =  \frac{5\pi}{6}  \:  \:  \: ou \:  \:  50 {}^{o} }

Na questão é dito que o vetor (u) faz um ângulo agudo com o vetor j, isto é, um ângulo menor que 90°. Observando os cálculos feitos, é possível ver que o vetor  \bf u_1 obteve um ângulo agudo com o vetor j.

Espero ter ajudado

Perguntas interessantes